number of crimes in city Y

This topic has expert replies
Senior | Next Rank: 100 Posts
Posts: 69
Joined: Mon Sep 15, 2008 7:01 pm
Thanked: 2 times

number of crimes in city Y

by ManSab » Thu Jul 16, 2009 7:05 pm
To decrease the number of crimes in city Y, the city's Police Commissioner proposed taking some police officers from low-crime districts of the city and moving them to high-crime districts of the city. His proposal is based on city Y crime data that show that the number of crimes in any district of the city decreases when additional police officers are moved into that district.

The Police Commissioner's proposal depends on which of the following assumptions?

1) City X experienced a drastic reduction in crime after implementing a proposal similar to that proposed by the Police Commissioner of city Y.

2) The severity of crimes committed in any district of the city decreases when additional police officers are moved into that district.

3)The number of crimes committed in all high-crime districts of city Y is more than triple the number of crimes committed in all low-crime districts of city Y.

4)There are more low-crime districts than high-crime districts in city Y.

5)Districts of the city from which police officers are removed do not experience significant crime increases shortly after the removal of those officers.

OA.. later
Please provide your answers with reasoning.

Thanks

Legendary Member
Posts: 527
Joined: Mon Jun 02, 2008 9:14 am
Location: Atlanta
Thanked: 17 times

by pandeyvineet24 » Thu Jul 16, 2009 7:16 pm
option E for me.

Senior | Next Rank: 100 Posts
Posts: 69
Joined: Mon Sep 15, 2008 7:01 pm
Thanked: 2 times

by ManSab » Thu Jul 16, 2009 7:24 pm
pandeyvineet24 - Could you please explain how you reached to this assumption.

E....is correct.

Thanks

User avatar
MBA Student
Posts: 1194
Joined: Sat Aug 16, 2008 9:42 pm
Location: Paris, France
Thanked: 71 times
Followed by:17 members
GMAT Score:710

by gmat740 » Thu Jul 16, 2009 7:41 pm
If you negate E, then it will weaken the argument

lets take a look
5)Districts of the city from which police officers are removed do not experience significant crime increases shortly after the removal of those officers.
Districts of the city from which police officers are removed experience significant crime increases shortly after the removal of those officers.

So clearly the move was detrimental. Thus this is the assumption.

Hope this helps

Master | Next Rank: 500 Posts
Posts: 171
Joined: Mon Jun 01, 2009 8:59 pm
Thanked: 8 times

E

by brick2009 » Thu Jul 16, 2009 7:48 pm
An assumption that has to be valid for the argument to hold good is:
If you reduce officers in one part of the city.... the crime in that part will not increase...

Also.. in the argument its is clearly stated that: High # of officers = Less crime...

Also simple elimination process:
a) City X? (out)
b.) no mention of severity in th argument... (out)
c.) Not abtout comparing number of crimes in Y..? (out)

d.) It is not abt number of lowe crime DISTRICTS? (out)
e.) Is the typical LMS (last man standing)..solution

Senior | Next Rank: 100 Posts
Posts: 69
Joined: Mon Sep 15, 2008 7:01 pm
Thanked: 2 times

by ManSab » Fri Jul 17, 2009 2:54 pm
thanks gmat740 and brick2009.

Legendary Member
Posts: 1169
Joined: Sun Jul 06, 2008 2:34 am
Thanked: 25 times
Followed by:1 members

by aj5105 » Sat Jul 18, 2009 10:04 am

Master | Next Rank: 500 Posts
Posts: 300
Joined: Fri Aug 09, 2013 2:26 am

by khanshainur » Sun May 15, 2016 11:42 pm
I feel the answer will be E

Senior | Next Rank: 100 Posts
Posts: 52
Joined: Tue Aug 05, 2014 3:20 am

by Eric77Gorm » Sun May 15, 2016 11:44 pm
I would go with option E as the correct option